LSAT and Law School Admissions Forum

Get expert LSAT preparation and law school admissions advice from PowerScore Test Preparation.

User avatar
 Dave Killoran
PowerScore Staff
  • PowerScore Staff
  • Posts: 5852
  • Joined: Mar 25, 2011
|
#26922
Complete Question Explanation
(The complete setup for this game can be found here: lsat/viewtopic.php?t=1572)

The correct answer choice is (A)


Answer choices (B), (C), (E) can each be eliminated because Y does not arrive fourth (there are also other reasons for eliminating (B), (C) and (E)). Answer choice (D) can be eliminated since T arrives ahead of Y, a violation of the second rule. Hence, answer choice (A) is correct.

Get the most out of your LSAT Prep Plus subscription.

Analyze and track your performance with our Testing and Analytics Package.